second theorem of Minkowski proof












1












$begingroup$


I am wondering if anyone have a proof of second theorem of Minkowski.
He says that if $vol(K) = 2^{n} det(L)$ and $K$ is compact and symmetric and convex then $K$ contains a non zero lattice point. I would appreciate your help.



Thanks!










share|cite|improve this question











$endgroup$












  • $begingroup$
    I think you need some more hypotheses on $K$.
    $endgroup$
    – Lord Shark the Unknown
    Dec 10 '18 at 21:05










  • $begingroup$
    yeah edited thanks, do you have an idea?@LordSharktheUnknown
    $endgroup$
    – user99812
    Dec 10 '18 at 21:09












  • $begingroup$
    And, what is $L$?
    $endgroup$
    – Berci
    Dec 10 '18 at 21:19










  • $begingroup$
    $L$ is a lattice @Berci
    $endgroup$
    – user99812
    Dec 10 '18 at 21:21
















1












$begingroup$


I am wondering if anyone have a proof of second theorem of Minkowski.
He says that if $vol(K) = 2^{n} det(L)$ and $K$ is compact and symmetric and convex then $K$ contains a non zero lattice point. I would appreciate your help.



Thanks!










share|cite|improve this question











$endgroup$












  • $begingroup$
    I think you need some more hypotheses on $K$.
    $endgroup$
    – Lord Shark the Unknown
    Dec 10 '18 at 21:05










  • $begingroup$
    yeah edited thanks, do you have an idea?@LordSharktheUnknown
    $endgroup$
    – user99812
    Dec 10 '18 at 21:09












  • $begingroup$
    And, what is $L$?
    $endgroup$
    – Berci
    Dec 10 '18 at 21:19










  • $begingroup$
    $L$ is a lattice @Berci
    $endgroup$
    – user99812
    Dec 10 '18 at 21:21














1












1








1





$begingroup$


I am wondering if anyone have a proof of second theorem of Minkowski.
He says that if $vol(K) = 2^{n} det(L)$ and $K$ is compact and symmetric and convex then $K$ contains a non zero lattice point. I would appreciate your help.



Thanks!










share|cite|improve this question











$endgroup$




I am wondering if anyone have a proof of second theorem of Minkowski.
He says that if $vol(K) = 2^{n} det(L)$ and $K$ is compact and symmetric and convex then $K$ contains a non zero lattice point. I would appreciate your help.



Thanks!







integer-lattices vector-lattices






share|cite|improve this question















share|cite|improve this question













share|cite|improve this question




share|cite|improve this question








edited Dec 10 '18 at 21:08







user99812

















asked Dec 10 '18 at 20:55









user99812user99812

62




62












  • $begingroup$
    I think you need some more hypotheses on $K$.
    $endgroup$
    – Lord Shark the Unknown
    Dec 10 '18 at 21:05










  • $begingroup$
    yeah edited thanks, do you have an idea?@LordSharktheUnknown
    $endgroup$
    – user99812
    Dec 10 '18 at 21:09












  • $begingroup$
    And, what is $L$?
    $endgroup$
    – Berci
    Dec 10 '18 at 21:19










  • $begingroup$
    $L$ is a lattice @Berci
    $endgroup$
    – user99812
    Dec 10 '18 at 21:21


















  • $begingroup$
    I think you need some more hypotheses on $K$.
    $endgroup$
    – Lord Shark the Unknown
    Dec 10 '18 at 21:05










  • $begingroup$
    yeah edited thanks, do you have an idea?@LordSharktheUnknown
    $endgroup$
    – user99812
    Dec 10 '18 at 21:09












  • $begingroup$
    And, what is $L$?
    $endgroup$
    – Berci
    Dec 10 '18 at 21:19










  • $begingroup$
    $L$ is a lattice @Berci
    $endgroup$
    – user99812
    Dec 10 '18 at 21:21
















$begingroup$
I think you need some more hypotheses on $K$.
$endgroup$
– Lord Shark the Unknown
Dec 10 '18 at 21:05




$begingroup$
I think you need some more hypotheses on $K$.
$endgroup$
– Lord Shark the Unknown
Dec 10 '18 at 21:05












$begingroup$
yeah edited thanks, do you have an idea?@LordSharktheUnknown
$endgroup$
– user99812
Dec 10 '18 at 21:09






$begingroup$
yeah edited thanks, do you have an idea?@LordSharktheUnknown
$endgroup$
– user99812
Dec 10 '18 at 21:09














$begingroup$
And, what is $L$?
$endgroup$
– Berci
Dec 10 '18 at 21:19




$begingroup$
And, what is $L$?
$endgroup$
– Berci
Dec 10 '18 at 21:19












$begingroup$
$L$ is a lattice @Berci
$endgroup$
– user99812
Dec 10 '18 at 21:21




$begingroup$
$L$ is a lattice @Berci
$endgroup$
– user99812
Dec 10 '18 at 21:21










1 Answer
1






active

oldest

votes


















0












$begingroup$

Let me take the slightly stronger condition $mathrm{vol}(K) > 2^n det(L)$, rather than $mathrm{vol}(K) = 2^n det(L)$. (You can then use the fact that $K$ is compact to argue that equality actually suffices.) By applying a linear transformation, let's also assume that $L$ is the integer lattice $mathbb{Z}^n$, so that we simply have $mathrm{vol}(K) > 2^n$. (This step isn't necessary at all, but I think it makes the proof easier to understand.)



For vectors $x,x' in mathbb{R}^n$, let's write $x equiv x' mathbb{Z}^n$ if $x - x' in mathbb{Z}^n$. (I.e. $x$ and $x'$ have coordinates with the same fractional part.) Suppose that there exists distinct $x, x' in K/2$ such that $x equiv x' bmod mathbb{Z}^n$. Then, $x - x' in mathbb{Z}^n$ by definition, and $x - x' neq 0$ because $x$ and $x'$ are distinct. However, by symmetry, $-x' in K/2$, and by convexity, $(x-x')/2 in K/2$ so that $x - x'$ is a non-zero lattice vector in $K$.



It therefore remains to find such an $x, x' in K/2$. To that end, we simply notice that for every $x in mathbb{R}^n$, there is a unique representative $f(x) equiv x bmod mathbb{Z}^n$ with $f(x) in [0,1)^n$. Notice that the map $x$ is a piecewise combination of volume-preserving maps. So, if $f$ is an injective mapping when restricted to $K/2$, then we must have
$$mathrm{vol}(f(K/2)) = mathrm{vol}(K/2) = 2^{-n}mathrm{vol}(K) > 1
; .
$$

But, this can't be true, since the entire image of $f$ is $[0,1)^n$, which has volume $1$. Therefore, $f$ is not injective over $K/2$. I.e., there exist distinct $x,x' in K/2$ such that $f(x) = f(x')$, as needed.



Edit: For what it's worth, this is typically referred to as Minkowski's first theorem. His second theorem finds a set of linearly independent vectors satisfying a certain constraint.






share|cite|improve this answer









$endgroup$













    Your Answer





    StackExchange.ifUsing("editor", function () {
    return StackExchange.using("mathjaxEditing", function () {
    StackExchange.MarkdownEditor.creationCallbacks.add(function (editor, postfix) {
    StackExchange.mathjaxEditing.prepareWmdForMathJax(editor, postfix, [["$", "$"], ["\\(","\\)"]]);
    });
    });
    }, "mathjax-editing");

    StackExchange.ready(function() {
    var channelOptions = {
    tags: "".split(" "),
    id: "69"
    };
    initTagRenderer("".split(" "), "".split(" "), channelOptions);

    StackExchange.using("externalEditor", function() {
    // Have to fire editor after snippets, if snippets enabled
    if (StackExchange.settings.snippets.snippetsEnabled) {
    StackExchange.using("snippets", function() {
    createEditor();
    });
    }
    else {
    createEditor();
    }
    });

    function createEditor() {
    StackExchange.prepareEditor({
    heartbeatType: 'answer',
    autoActivateHeartbeat: false,
    convertImagesToLinks: true,
    noModals: true,
    showLowRepImageUploadWarning: true,
    reputationToPostImages: 10,
    bindNavPrevention: true,
    postfix: "",
    imageUploader: {
    brandingHtml: "Powered by u003ca class="icon-imgur-white" href="https://imgur.com/"u003eu003c/au003e",
    contentPolicyHtml: "User contributions licensed under u003ca href="https://creativecommons.org/licenses/by-sa/3.0/"u003ecc by-sa 3.0 with attribution requiredu003c/au003e u003ca href="https://stackoverflow.com/legal/content-policy"u003e(content policy)u003c/au003e",
    allowUrls: true
    },
    noCode: true, onDemand: true,
    discardSelector: ".discard-answer"
    ,immediatelyShowMarkdownHelp:true
    });


    }
    });














    draft saved

    draft discarded


















    StackExchange.ready(
    function () {
    StackExchange.openid.initPostLogin('.new-post-login', 'https%3a%2f%2fmath.stackexchange.com%2fquestions%2f3034453%2fsecond-theorem-of-minkowski-proof%23new-answer', 'question_page');
    }
    );

    Post as a guest















    Required, but never shown

























    1 Answer
    1






    active

    oldest

    votes








    1 Answer
    1






    active

    oldest

    votes









    active

    oldest

    votes






    active

    oldest

    votes









    0












    $begingroup$

    Let me take the slightly stronger condition $mathrm{vol}(K) > 2^n det(L)$, rather than $mathrm{vol}(K) = 2^n det(L)$. (You can then use the fact that $K$ is compact to argue that equality actually suffices.) By applying a linear transformation, let's also assume that $L$ is the integer lattice $mathbb{Z}^n$, so that we simply have $mathrm{vol}(K) > 2^n$. (This step isn't necessary at all, but I think it makes the proof easier to understand.)



    For vectors $x,x' in mathbb{R}^n$, let's write $x equiv x' mathbb{Z}^n$ if $x - x' in mathbb{Z}^n$. (I.e. $x$ and $x'$ have coordinates with the same fractional part.) Suppose that there exists distinct $x, x' in K/2$ such that $x equiv x' bmod mathbb{Z}^n$. Then, $x - x' in mathbb{Z}^n$ by definition, and $x - x' neq 0$ because $x$ and $x'$ are distinct. However, by symmetry, $-x' in K/2$, and by convexity, $(x-x')/2 in K/2$ so that $x - x'$ is a non-zero lattice vector in $K$.



    It therefore remains to find such an $x, x' in K/2$. To that end, we simply notice that for every $x in mathbb{R}^n$, there is a unique representative $f(x) equiv x bmod mathbb{Z}^n$ with $f(x) in [0,1)^n$. Notice that the map $x$ is a piecewise combination of volume-preserving maps. So, if $f$ is an injective mapping when restricted to $K/2$, then we must have
    $$mathrm{vol}(f(K/2)) = mathrm{vol}(K/2) = 2^{-n}mathrm{vol}(K) > 1
    ; .
    $$

    But, this can't be true, since the entire image of $f$ is $[0,1)^n$, which has volume $1$. Therefore, $f$ is not injective over $K/2$. I.e., there exist distinct $x,x' in K/2$ such that $f(x) = f(x')$, as needed.



    Edit: For what it's worth, this is typically referred to as Minkowski's first theorem. His second theorem finds a set of linearly independent vectors satisfying a certain constraint.






    share|cite|improve this answer









    $endgroup$


















      0












      $begingroup$

      Let me take the slightly stronger condition $mathrm{vol}(K) > 2^n det(L)$, rather than $mathrm{vol}(K) = 2^n det(L)$. (You can then use the fact that $K$ is compact to argue that equality actually suffices.) By applying a linear transformation, let's also assume that $L$ is the integer lattice $mathbb{Z}^n$, so that we simply have $mathrm{vol}(K) > 2^n$. (This step isn't necessary at all, but I think it makes the proof easier to understand.)



      For vectors $x,x' in mathbb{R}^n$, let's write $x equiv x' mathbb{Z}^n$ if $x - x' in mathbb{Z}^n$. (I.e. $x$ and $x'$ have coordinates with the same fractional part.) Suppose that there exists distinct $x, x' in K/2$ such that $x equiv x' bmod mathbb{Z}^n$. Then, $x - x' in mathbb{Z}^n$ by definition, and $x - x' neq 0$ because $x$ and $x'$ are distinct. However, by symmetry, $-x' in K/2$, and by convexity, $(x-x')/2 in K/2$ so that $x - x'$ is a non-zero lattice vector in $K$.



      It therefore remains to find such an $x, x' in K/2$. To that end, we simply notice that for every $x in mathbb{R}^n$, there is a unique representative $f(x) equiv x bmod mathbb{Z}^n$ with $f(x) in [0,1)^n$. Notice that the map $x$ is a piecewise combination of volume-preserving maps. So, if $f$ is an injective mapping when restricted to $K/2$, then we must have
      $$mathrm{vol}(f(K/2)) = mathrm{vol}(K/2) = 2^{-n}mathrm{vol}(K) > 1
      ; .
      $$

      But, this can't be true, since the entire image of $f$ is $[0,1)^n$, which has volume $1$. Therefore, $f$ is not injective over $K/2$. I.e., there exist distinct $x,x' in K/2$ such that $f(x) = f(x')$, as needed.



      Edit: For what it's worth, this is typically referred to as Minkowski's first theorem. His second theorem finds a set of linearly independent vectors satisfying a certain constraint.






      share|cite|improve this answer









      $endgroup$
















        0












        0








        0





        $begingroup$

        Let me take the slightly stronger condition $mathrm{vol}(K) > 2^n det(L)$, rather than $mathrm{vol}(K) = 2^n det(L)$. (You can then use the fact that $K$ is compact to argue that equality actually suffices.) By applying a linear transformation, let's also assume that $L$ is the integer lattice $mathbb{Z}^n$, so that we simply have $mathrm{vol}(K) > 2^n$. (This step isn't necessary at all, but I think it makes the proof easier to understand.)



        For vectors $x,x' in mathbb{R}^n$, let's write $x equiv x' mathbb{Z}^n$ if $x - x' in mathbb{Z}^n$. (I.e. $x$ and $x'$ have coordinates with the same fractional part.) Suppose that there exists distinct $x, x' in K/2$ such that $x equiv x' bmod mathbb{Z}^n$. Then, $x - x' in mathbb{Z}^n$ by definition, and $x - x' neq 0$ because $x$ and $x'$ are distinct. However, by symmetry, $-x' in K/2$, and by convexity, $(x-x')/2 in K/2$ so that $x - x'$ is a non-zero lattice vector in $K$.



        It therefore remains to find such an $x, x' in K/2$. To that end, we simply notice that for every $x in mathbb{R}^n$, there is a unique representative $f(x) equiv x bmod mathbb{Z}^n$ with $f(x) in [0,1)^n$. Notice that the map $x$ is a piecewise combination of volume-preserving maps. So, if $f$ is an injective mapping when restricted to $K/2$, then we must have
        $$mathrm{vol}(f(K/2)) = mathrm{vol}(K/2) = 2^{-n}mathrm{vol}(K) > 1
        ; .
        $$

        But, this can't be true, since the entire image of $f$ is $[0,1)^n$, which has volume $1$. Therefore, $f$ is not injective over $K/2$. I.e., there exist distinct $x,x' in K/2$ such that $f(x) = f(x')$, as needed.



        Edit: For what it's worth, this is typically referred to as Minkowski's first theorem. His second theorem finds a set of linearly independent vectors satisfying a certain constraint.






        share|cite|improve this answer









        $endgroup$



        Let me take the slightly stronger condition $mathrm{vol}(K) > 2^n det(L)$, rather than $mathrm{vol}(K) = 2^n det(L)$. (You can then use the fact that $K$ is compact to argue that equality actually suffices.) By applying a linear transformation, let's also assume that $L$ is the integer lattice $mathbb{Z}^n$, so that we simply have $mathrm{vol}(K) > 2^n$. (This step isn't necessary at all, but I think it makes the proof easier to understand.)



        For vectors $x,x' in mathbb{R}^n$, let's write $x equiv x' mathbb{Z}^n$ if $x - x' in mathbb{Z}^n$. (I.e. $x$ and $x'$ have coordinates with the same fractional part.) Suppose that there exists distinct $x, x' in K/2$ such that $x equiv x' bmod mathbb{Z}^n$. Then, $x - x' in mathbb{Z}^n$ by definition, and $x - x' neq 0$ because $x$ and $x'$ are distinct. However, by symmetry, $-x' in K/2$, and by convexity, $(x-x')/2 in K/2$ so that $x - x'$ is a non-zero lattice vector in $K$.



        It therefore remains to find such an $x, x' in K/2$. To that end, we simply notice that for every $x in mathbb{R}^n$, there is a unique representative $f(x) equiv x bmod mathbb{Z}^n$ with $f(x) in [0,1)^n$. Notice that the map $x$ is a piecewise combination of volume-preserving maps. So, if $f$ is an injective mapping when restricted to $K/2$, then we must have
        $$mathrm{vol}(f(K/2)) = mathrm{vol}(K/2) = 2^{-n}mathrm{vol}(K) > 1
        ; .
        $$

        But, this can't be true, since the entire image of $f$ is $[0,1)^n$, which has volume $1$. Therefore, $f$ is not injective over $K/2$. I.e., there exist distinct $x,x' in K/2$ such that $f(x) = f(x')$, as needed.



        Edit: For what it's worth, this is typically referred to as Minkowski's first theorem. His second theorem finds a set of linearly independent vectors satisfying a certain constraint.







        share|cite|improve this answer












        share|cite|improve this answer



        share|cite|improve this answer










        answered Dec 14 '18 at 19:41









        Noah Stephens-DavidowitzNoah Stephens-Davidowitz

        1636




        1636






























            draft saved

            draft discarded




















































            Thanks for contributing an answer to Mathematics Stack Exchange!


            • Please be sure to answer the question. Provide details and share your research!

            But avoid



            • Asking for help, clarification, or responding to other answers.

            • Making statements based on opinion; back them up with references or personal experience.


            Use MathJax to format equations. MathJax reference.


            To learn more, see our tips on writing great answers.




            draft saved


            draft discarded














            StackExchange.ready(
            function () {
            StackExchange.openid.initPostLogin('.new-post-login', 'https%3a%2f%2fmath.stackexchange.com%2fquestions%2f3034453%2fsecond-theorem-of-minkowski-proof%23new-answer', 'question_page');
            }
            );

            Post as a guest















            Required, but never shown





















































            Required, but never shown














            Required, but never shown












            Required, but never shown







            Required, but never shown

































            Required, but never shown














            Required, but never shown












            Required, but never shown







            Required, but never shown







            Popular posts from this blog

            Bundesstraße 106

            Le Mesnil-Réaume

            Ida-Boy-Ed-Garten